LSAT and Law School Admissions Forum

Get expert LSAT preparation and law school admissions advice from PowerScore Test Preparation.

 Administrator
PowerScore Staff
  • PowerScore Staff
  • Posts: 8929
  • Joined: Feb 02, 2011
|
#28486
Complete Question Explanation
(The complete setup for this game can be found here: lsat/viewtopic.php?t=9894)

The correct answer choice is (D)

The question stem adds the following condition:
  • ..... ..... ..... ..... K :longline: H :longline: J

Because template #2 specifies that J :longline: K, template #2 cannot apply, and only template #1 is applicable. Adding the question stem condition to template #1 produces the following diagram:
Dec 06_M12 game #2_cr_game#2_#10_diagram_1.png
Consequently, K is the most expensive suite and H is the second most expensive suite.

Answer choice (A): This answer choice is incorrect because F cannot be more expensive than H according to the diagram above.

Answer choice (B): This answer choice is incorrect because G cannot be more expensive than F according to the diagram above.

Answer choice (C): This answer choice is incorrect because G cannot be more expensive than H according to the diagram above.

Answer choice (D): This is the correct answer choice. J can be more expensive than L.

Answer choice (E): This answer choice is incorrect because L cannot be more expensive than K according to the diagram above.

Overall, this game is made easier if you use the last rule to create two super-sequence templates. If you do not recognize how the game is controlled by the templates, this game can be very tricky and time-consuming.
You do not have the required permissions to view the files attached to this post.

Get the most out of your LSAT Prep Plus subscription.

Analyze and track your performance with our Testing and Analytics Package.